Đến nội dung

Hình ảnh

$Cho a,b,c\geq 0 \sum a\doteq 1 \sum \sqrt{\frac{a}{2a^{2}+bc}}\geq 2$

bdt

  • Please log in to reply
Chủ đề này có 2 trả lời

#1
TARGET

TARGET

    Binh nhì

  • Thành viên mới
  • 17 Bài viết

Chứng minh với $ a,b,c\geq 0 \sum a\doteq 1:\sum \sqrt{\frac{a}{2a^{2}+bc}}\geq 2$


Bài viết đã được chỉnh sửa nội dung bởi TARGET: 07-03-2022 - 23:58

$\sqrt[5]{\frac{a^{5}+b^{5}}{2}}\doteq \sqrt[5]{\frac{a^{5}+b^{5}}{a^{4}+b^{4}}\frac{a^{4}+b^{4}}{a^{3}+b^{3}}\frac{a^{3}+b^{3}}{a^{2}+b^{2}}\frac{a^{2}+b^{2}}{a+b}\frac{a+b}{2}}$


#2
KietLW9

KietLW9

    Đại úy

  • Điều hành viên THCS
  • 1737 Bài viết

Đặt $a+b+c=p;ab+bc+ca=q;abc=r$ thì ta chú ý tới đẳng thức sau: $(a-b)^2(b-c)^2(c-a)^2=p^2q^2 - 4q^3 + 2p(9q - 2p^2)r - 27r^2=q^2-4q^3+2(9q-2)r-27r^2$

Áp dụng bất đẳng thức AM - GM, ta được: $\sqrt{\frac{a}{2a^2+bc}}=\frac{a}{\sqrt{(a^2+ab+ac)(2a^2+bc)}}\geqslant \frac{2a}{3a^2+ab+bc+ca}$

Ta quy về chứng minh: $\frac{a}{3a^2+ab+bc+ca}+\frac{b}{3b^2+ab+bc+ca}+\frac{c}{3c^2+ab+bc+ca}\geqslant 1$

$\Leftrightarrow 9abc(ab+bc+ca)+3(ab+bc+ca)\left [ ab(a+b)+bc(b+c)+ca(c+a) \right ]+(ab+bc+ca)^2(a+b+c)\geqslant 27a^2b^2c^2+9(ab+bc+ca)(a^2b^2+b^2c^2+c^2a^2)+3(ab+bc+ca)^2(a^2+b^2+c^2)+(ab+bc+ca)^3$

$\Leftrightarrow 9qr+3q(pq-3r)+q^2p\geqslant 27r^2+9q(q^2-2pr)+3q^2(p^2-2q)+q^3$

$\Leftrightarrow 9qr+3q(q-3r)+q^2\geqslant 27r^2+9q(q^2-2r)+3q^2(1-2q)+q^3\Leftrightarrow \left [ q^2-4q^3+2(9q-2)r-27r^2 \right ]+4r\geqslant 0$

Suy cho cùng thì bất đẳng thức tương đương: $(a-b)^2(b-c)^2(c-a)^2+4abc\geqslant 0$

Bất đẳng thức được chứng minh

Dấu bằng xảy ra khi $a=b=\frac{1}{2},c=0$ hoặc các hoán vị


Trong cuộc sống không có gì là đẳng thức , tất cả đều là bất đẳng thức  :ukliam2:   :ukliam2: 

 

 

$\text{LOVE}(\text{KT}) S_a (b - c)^2 + S_b (c - a)^2 + S_c (a - b)^2 \geqslant 0\forall S_a,S_b,S_c\geqslant 0$

 

 

 


#3
TARGET

TARGET

    Binh nhì

  • Thành viên mới
  • 17 Bài viết

Đặt $a+b+c=p;ab+bc+ca=q;abc=r$ thì ta chú ý tới đẳng thức sau: $(a-b)^2(b-c)^2(c-a)^2=p^2q^2 - 4q^3 + 2p(9q - 2p^2)r - 27r^2=q^2-4q^3+2(9q-2)r-27r^2$

Áp dụng bất đẳng thức AM - GM, ta được: $\sqrt{\frac{a}{2a^2+bc}}=\frac{a}{\sqrt{(a^2+ab+ac)(2a^2+bc)}}\geqslant \frac{2a}{3a^2+ab+bc+ca}$

Ta quy về chứng minh: $\frac{a}{3a^2+ab+bc+ca}+\frac{b}{3b^2+ab+bc+ca}+\frac{c}{3c^2+ab+bc+ca}\geqslant 1$

$\Leftrightarrow 9abc(ab+bc+ca)+3(ab+bc+ca)\left [ ab(a+b)+bc(b+c)+ca(c+a) \right ]+(ab+bc+ca)^2(a+b+c)\geqslant 27a^2b^2c^2+9(ab+bc+ca)(a^2b^2+b^2c^2+c^2a^2)+3(ab+bc+ca)^2(a^2+b^2+c^2)+(ab+bc+ca)^3$

$\Leftrightarrow 9qr+3q(pq-3r)+q^2p\geqslant 27r^2+9q(q^2-2pr)+3q^2(p^2-2q)+q^3$

$\Leftrightarrow 9qr+3q(q-3r)+q^2\geqslant 27r^2+9q(q^2-2r)+3q^2(1-2q)+q^3\Leftrightarrow \left [ q^2-4q^3+2(9q-2)r-27r^2 \right ]+4r\geqslant 0$

Suy cho cùng thì bất đẳng thức tương đương: $(a-b)^2(b-c)^2(c-a)^2+4abc\geqslant 0$

Bất đẳng thức được chứng minh

Dấu bằng xảy ra khi $a=b=\frac{1}{2},c=0$ hoặc các hoán vị

WOW bổ đề mạnh vậy

Một cách khác p,q,r mà không cần dùng bổ đề

Với c=0

$\sqrt{\frac{1}{2a}}+\sqrt{\frac{1}{2b}}\geq 2$(Luôn đúng với AM-GM)

Với $c\neq 0$

Ta cần Cm vế trái luôn $> 2$.Thật vậy bình phương lên ta có như sau

VT=$\sum \frac{a}{2a^{2}+bc} +2\sqrt{\frac{ab}{\left (2a^{2}+bc\right)\left ( 2b^{2}+ca \right )}}> 4$

 

$\left ( \doteq \right )$$\sum \frac{\left ( a+b \right )\left ( a+c \right )}{2a^{2}+bc} +\sum \frac{2a^{2}}{2a^{2}+bc} +2\sum \sqrt{\frac{ab}{\left ( 2a^{2}+bc \right )\left ( 2b^{2}+ca \right )}}> 7$

Từ đây ta có $\sum \frac{\left ( a+b \right )\left ( a+c \right )}{2a^{2}+bc}= \left ( a+b \right )\left ( a+c \right )\left ( b+c \right )\left ( \sum \frac{1}{\left ( 2a^{2}+bc \right )\left ( b+c \right )} \right ) \geq 3\frac{\left ( a+b \right )\left ( b+c \right )\left ( c+a \right )}{\sum ab\left ( a+b \right )} \doteq 3\frac{q-r}{q-3r}$

$\sum \frac{2a^{2}}{2a^{2}+bc}\geq \frac{2}{2-3q}$(Theo C-S)

Mà từ a+b+c=1 suy ra

$ab\leq \frac{1}{4}$ tương tự với ac và bc thì ta có:

$2\sum \sqrt{\frac{ab}{\left ( 2a^{2}+bc \right )\left ( 2b^{2} +ca\right )}}\doteq 2\sum \frac{ab}{\sqrt{2a^{2}b+b^{2}c}\sqrt{2b^{2}a+ca^{2}}}\geq 16\frac{q^{2}}{3pq-9r}$(Theo C-S và AM-GM)

$\left ( \doteq \right )r\left ( 36-54q \right ) +\frac{52}{3}q^{2}-6q> 0 \left ( \doteq \right )\frac{\left ( 4q-1 \right )\left ( 36-54q \right )}{9}+\frac{52}{3}q^{2}-6q> 0 \left ( \doteq \right )5q\left ( q-12 \right )+3<0$

Luôn đúng với $0\leq q\leq \frac{1}{3}$


Bài viết đã được chỉnh sửa nội dung bởi TARGET: 08-03-2022 - 11:50

$\sqrt[5]{\frac{a^{5}+b^{5}}{2}}\doteq \sqrt[5]{\frac{a^{5}+b^{5}}{a^{4}+b^{4}}\frac{a^{4}+b^{4}}{a^{3}+b^{3}}\frac{a^{3}+b^{3}}{a^{2}+b^{2}}\frac{a^{2}+b^{2}}{a+b}\frac{a+b}{2}}$






Được gắn nhãn với một hoặc nhiều trong số những từ khóa sau: bdt

1 người đang xem chủ đề

0 thành viên, 1 khách, 0 thành viên ẩn danh